An item is regularly priced at $80. Chau bought it on sale for 15% off the regular price. How much did Chau pay?

Answers

Answer 1
15% of $80 = 15*80/100 = $12
$80 - $12 = $68
Chau pay $68
Answer 2

only $68...steps.... (80*15% =12 /80-12=68


Related Questions

The rectangle has a length of 4x+3 and a width of 3x. Show your work.

A. Find the perimeter.


B. Find the area.


C. Find the perimeter and area if x = 8.

Answers

Answer:

  A.  P = 14x+6

  B.  A = 12x^2 +9x

  C.  P = 118; A = 840

Step-by-step explanation:

A. The perimeter is twice the sum of length and width:

  P = 2(L +W) = 2((4x+3) +(3x)) = 2(7x +3)

  P = 14x +6 . . . . the perimeter of the rectangle

__

B. The area is the product of length and width:

  A = LW = (4x +3)(3x)

  A = 12x^2 +9x . . . . . the area of the rectangle

__

C. When x = 8, these values are ...

  P = 14·8 +6 = 118 . . . . . perimeter in units

  A = 12·8^2 +9·8 = 768 +72 = 840 . . . . . area in square units

Answer:

a) [tex] P = 2(4x+3) +2(3x) =8x +6 +6x = 14x +6[/tex]

b) [tex] A= 12x^2 +9x[/tex]

c) [tex] P = 14*8 +6 = 112+6 = 118[/tex]

[tex] A= 12(8)^2 +9*8 = 840[/tex]

Step-by-step explanation:

We know that the length is 4x+3 and the width is of 3x

Part a

For this case the perimeter is given by:

[tex] P = 2(4x+3) +2(3x) =8x +6 +6x = 14x +6[/tex]

Part b

The area is given by:

[tex] A= (4x+3) (3x)[/tex]

And after multiply we got:

[tex] A= 12x^2 +9x[/tex]

Part c

For this case replacing the value of x =8 we got:

[tex] P = 14*8 +6 = 112+6 = 118[/tex]

[tex] A= 12(8)^2 +9*8 = 840[/tex]

line a passes through (0,3) & (-4,8) line b passes through (0,5) & (5,9) lines a & b are?

Answers

Answer:

line a )y=-5/4x+3

line b) y=4/5+5

Adam works for an agency

Answers

Well that isn’t really a question but good for him

The composite figure shown is made up of a cylinder and a___a0.

Answers

Answer: Cone

Step-by-step explanation:

Looking at the shape, the problem already listed cylinder. That is the bottom figure. The top figure is a cone.

Answer: cone

Step-by-step explanation:

The bottom figure is a cylinder. A cylinder is basically a rectangle wrapped around two circles.

The top figure is a cone. A cone’s net looks like a triangle with a curved base wrapped around a circle. If you don’t believe the top figure is a cone, think about what ice cream cones look like. The top figure is a cone.

Pls hit the thx button and mark me brainliest if this helped :)

Natasha's sculpture was 5 3/16 inches tall. Mayas was 4 times as tall.How much shorter was Natasha's sculpture than Mayas​

Answers

Answer:

Natasha's sculpture is 249/16 or 15.5625 or 15 9/16  inches shorter than Mayas sculpture

Step-by-step explanation:

The height of Natasha sculpture is 5 3/16 inches tall. Let us convert the height to an improper fraction.

5 3/16 = 83/16 inches tall. Therefore,

Natasha's sculpture is 83/16 inches tall.

According to the question Mayas own sculpture was 4 times as tall as Natasha's own sculpture. Mathematically, Mayas sculpture height can be expressed as follows

Mayas sculpture height = 4(83/16)

Mayas sculpture height = 4 × 83/16

Mayas sculpture height = 332/16

Mayas sculpture height = 20.75  inches or 83/4 inches

To know how much shorter was Natasha's sculpture we subtract Natasha sculpture height from Mayas sculpture height.

Therefore,

83/4 - 83/16 = (332 - 83) /16 = 249/16 or 15.5625 or 15 9/16

Natasha's sculpture is 249/16 or 15.5625 or 15 9/16  inches shorter than Mayas sculpture

In the flowering plant, white flowers (B) are dominant over red flowers (b), and short plants (E) are dominant over tall flower (e). An F2 generation was created by crossing two F1 individuals (each BbEe). The data below are the counted number of flowers from each phenotype in the F2 generation.

White short= 206
Red, short= 83
White, tall= 85
Red, tall= 30

Required:
a. What is your null hypothesis?
b. What is your expected phenotypic ratio based on Mendelian inheritance?

Answers

Question Completion

(a)What is your null hypothesis?

(b)What is your expected phenotypic ratio based on Mendelian inheritance?

(c)Calculate the expected number of flowers you should have gotten based on the Mendelian inheritance. Then calculate a chi-square value, degrees of freedom, and a p-value.

Chi-square statistic: _____Degrees of freedom (# phenotypes -1): P-value:

(d)Interpret your results. Do you reject it or fail to reject your null hypothesis (please restate the null)?

Answer:

(a)[tex]H_0:$The given data fit the predicted phenotype[/tex]

(b)9:3:3:1

(c)

Chi-square statistic: 3.8914Degrees of freedom (# phenotypes -1) =3P-value:  0.2734

(d)We fail to reject the null hypothesis.

Step-by-step explanation:

In the flowering plant, white flowers (B) are dominant over red flowers (b), and short plants (E) are dominant over tall flowers (e). An F2 generation was created by crossing two F1 individuals (each BbEe).

(a)The null hypothesis is:

[tex]H_0:$The given data fit the predicted phenotype[/tex]

(b)The gametes are BE, Be, bE and be.

The offsprings are presented in the table below:

[tex]\left|\begin{array}{c|cccc}&BE&Be&bE&be\\--&--&--&--&--\\BE&BE&BE&BE&BE\\Be&BE&Be&BE&Be\\bE&BE&BE&bE&bE\\be&BE&Be&bE&be\end{array}\right|[/tex]

The expected phenotypic ratio based on Mendelian inheritance

BE:Be:bE:be=9:3:3:1

(c)

[tex]\left|\begin{array}{c|c|c|c|c|c}$Phenotype&Observed&$Expected&O-E&(O-E)^2&\dfrac{(O-E)^2}{E} \\-----&--&--&--&--&--\\$White short(BE)&206&\frac{9}{16}*404 \approx 227 &-21&441&1.9427\\$Red, short(bE)&83&\frac{3}{16}*404 \approx 78 &5&25&0.3205\\$White, tall(Be)&85&\frac{3}{16}*404 \approx 78 &7&49&0.6282\\$Red, tall(be)&30&\frac{1}{16}*404 \approx 25 &5&25&1\\-----&--&--&--&--&--\\$Total&404&--&--&--&3.8914\end{array}\right|[/tex]

Therefore:

Chi-square statistic: 3.8914Degrees of freedom (# phenotypes -1):  4-1 =3P-value:  0.2734

(d) Our null hypothesis is:

[tex]H_0:$The given data fit the predicted phenotype[/tex]

Since p>0.05, the given data fit the predicted phenotypic ratio.

We, therefore, fail to reject the null hypothesis.

The difference in the observed and expected are sosmall that they can be attributed to random chance.

A family uses 12,986.64 Swiss francs per year to pay a mortgage that requires US dollars. Approximately how much, in US dollars, does the family spend per month on the mortgage?
1 US dollar = 0.9019 Swiss francs
1 Swiss franc = 1.11 US dollar
A.)$975
B.)$1,080
C.)$1,200
D.)$1,440

Answers

Answer:

C

Step-by-step explanation:

Per year: 12,986.64 * 1.11 = 14,415.1704 US dollars

Per month: 14,415.1704/12 = 1,201.2642 US dollars

Answer: c 1,200

explanation:  1,200 ÷ 12,986.64 = 0.09240265

How many cups of pineapple juice are needed to make 6 gallons of punch

Answers

Answer:

17

Step-by-step explanation:

2 Points
What is the value of y? 2y 50 y+10

Answers

Answer:

y= -5/24

Step-by-step explanation:

2y=50y+10

1: Subtract 50y from both sides.

2y−50y=50y+10−50y

−48y=10

2: Divide both sides by -48.

−48y /48y=10/-48

y= -5/24

A new car is purchased for 25000 dollars.The value of the car depreciates at 8.25% per year.What will the value of the car be,to the nearest cent,after 15 years?

Answers

Answer:

da,

Step-by-step explanation:

Solve the two-step equation. -9x + 0.4 = 4 Which operation must be performed to move all the constants to the right side of the equation? Then, which operation must be performed to isolate the variable? The solution to the equation is x = .

Answers

Answer:

x = -0.4

Step-by-step explanation:

We have the equation:

-9x + 0.4 = 4

First, the operation to move all the constants to the right side is subtraction since we would have to subtract 0.4 from each side, let's see this:

[tex]-9x+0.4=4\\-9x+0.4-0.4=4-0.4\\-9x=3.6[/tex]

Now, we have all the constants on the right side of the equation.

Now, the operation we need to perform to isolate the variable is division (since the x has a -9 that is being multiplied by x) we need to do the opposite operation:

[tex]-9x=3.6\\\frac{-9x}{-9} =\frac{3.6}{-9} \\x=-0.4[/tex]

Thus, the answer to this equation is x= -0.4

Answer:

1.) Which operation must be performed to move all the constants to the right side of the equation?

✔ Subtract 0.4 (C)

2.) Then, which operation must be performed to isolate the variable?

✔ Divide by -9 on both sides.  (D)

3.) The solution to the equation is x =

✔ -0.4   (B)

Step-by-step explanation:

I hope this helps!! Have a wonderful day!! :))

Which product is negative? (Negative StartFraction 3 over 8 EndFraction) (Negative StartFraction 5 over 7 EndFraction) (one-fourth) (StartFraction 3 over 8 EndFraction) (Negative StartFraction 5 over 7 EndFraction) (Negative one-fourth) (StartFraction 3 over 8 EndFraction) (StartFraction 5 over 7 EndFraction) (one-fourth) (Negative StartFraction 3 over 8 EndFraction) (Negative StartFraction 5 over 7 EndFraction) (negative one-fourth)

Answers

Answer:

[tex](D)\left(-\dfrac38\right)\left(-\dfrac57\right)\left(-\dfrac14\right)[/tex]

Step-by-step explanation:

The given options are:

[tex](A)\left(-\dfrac38\right)\left(-\dfrac57\right)\left(\dfrac14\right)\\(B)\left(\dfrac38\right)\left(-\dfrac57\right)\left(-\dfrac14\right)\\(C)\left(\dfrac38\right)\left(\dfrac57\right)\left(\dfrac14\right)\\(D)\left(-\dfrac38\right)\left(-\dfrac57\right)\left(-\dfrac14\right)[/tex]

The key to determining which product is negative is to understand the rule of sign multiplication.

Now:

The product of even negative terms is positiveThe product of odd negative terms is negative.The product of positive will always be positive.

In Options A and B, the number of negative signs is even, therefore our result is positive.

In option C, all the terms are positive, therefore our result will be positive.

In Option D, the number of negative signs is odd, therefore our result is negative.

Answer:

your answer is D

:)

Solve the formula for c

m=h/c-w

Answers

Answer:

c = h/(m+w)

Step-by-step explanation:

m=(h/c)-w

Add w to each side

m+w=h/c-w+w

m+w = h/c

Multiply each side by c

c(m+w) = h/c*c

c(m+w) = h

Divide each side by (m+w)

c(m+w)/(m+w) = h/(m+w)

c = h/(m+w)

Does anyone know the answer?

Answers

Answer:

5.44

Step-by-step explanation:

From Trigonometry Identity,

Sin65= CB/AB

CB = AB Sin65

= 6 × Sin65

= 5.4378

= 5.44{ to the nearest hundredth}

The goal is to find the length of side BC. You need the trigonometric function “sine” to solve this.

sin(angle) = (opposite) / (hypotenuse)

Plugging in:
sin(65°) = (?) / (6)

Solve for the (?):
6 * sin(65°) = (?)

With a calculator, I find the answer to be approximately
(?) = 0.90631

( 3/4 ÷ 3/100 −23 1/2 )÷1 1/2 · 2/3 +1 1/6
All of the (/) are fractions. Sorry if it's unclear.

Answers

Answer:

Simplify:  = −7333 4400 (Decimal: -1.666591)

Answer: −7333/  4400

Step-by-step explanation:

Steps below

1: 343100−2312112(2)3+116

2: 14100−2312112(2)3+116

3: 1400−2312112(2)3+116

4: −46199400112(2)3+116

Answers:

Simplify:  = −7333 4400 (Decimal: -1.666591)

Answer: −7333/  4400

Hope this helps.

Answer:

The answer is 11/6

Point D' is the image of D(-2, 1) under a reflection across the x-axis.

Answers

Answer:

-2, -1

Step-by-step explanation:

the x same cause if you reflect across the x-axis so you move on the y-axis. and reflect means the same distance from the point you reflect (that it was 1 cause it was 1 point above the x-axis) just negative so -1 now. if the original point was -2,-1 so the answer was -2,1.

What’s the correct answer for this?

Answers

。☆✼★ ━━━━━━━━━━━━━━  ☾  

Tangents that meet at a point are equal in length so JL and LM are equal

Let's form an equation:

3x + 10 = 7x - 6

+6 to both sides

3x + 16 = 7x

-3x from both sides

16 = 4x

/4 on both sides

x = 4

Now, we can sub this value into the equation:

7(4) - 6 = 22

Thus, your answer is option B. 22

Have A Nice Day ❤    

Stay Brainly! ヅ    

- Ally ✧    

。☆✼★ ━━━━━━━━━━━━━━  ☾

Answer:

2ND OPTION  

Step-by-step explanation:

in a circle , tangents drawn from an external point to the circle are equal.

ie JL = LM

3x+10 = 7x-6

10 + 6 = 7x- 3x

16 = 4x

x = 16/4

x=4

therefore LM = 7x -6 = 7*4 - 6

LM = 28 - 6 =22

therefore LM =22

HOPE IT HELPS. PLEASE MARK ME AS THE BRAINLIEST....

Daley went to the grocery store to buy berries. Blueberries cost $1.50 a pound and strawberries cost $2.25 a pound. Daley spent $10.50 on fruit. The equation he wrote to represent the scenario is 1.5b + 2.25s = 10.5. His solution is: 1. Subtract 2.25s: 1.5b = 10.5 − 2.25s 2. Divide by 1.5: 1.5b 1.5 = 10.5 − 2.25s 1.5 3. Distribute 1.5: b = 7 − 1.5s Daley wrote an equivalent equation to determine how many pounds of blueberries he could buy. Analyze Daley’s work. Did he make an error? Yes. In step 1, he had to subtract 1.5b from both sides. Yes. In step 2, he had to multiply 1.5 to both sides. Yes. In step 3, he can only multiply the 1.5 and 12.5 together. No. He correctly solved for b.

Answers

Answer: 1.5(b)+ 2.25(s) = 10.05

Step-by-step explanation:

Answer:

B & S

Step-by-step explanation:

According to R.L. Polk & Co., the average age of cars and light trucks in the U.S. is 10.8 years. Assume that the standard deviation for this population is 3.7 years. A sample of 35 randomly selected vehicles was selected. What is the probability that the sample mean will be greater than 10 years?

Answers

Answer:

10.03% probability that the sample mean will be greater than 10 years

Step-by-step explanation:

To solve this question, we need to understand the normal probability distribution and the central limit theorem.

Normal probability distribution

Problems of normally distributed samples are solved using the z-score formula.

In a set with mean [tex]\mu[/tex] and standard deviation [tex]\sigma[/tex], the zscore of a measure X is given by:

[tex]Z = \frac{X - \mu}{\sigma}[/tex]

The Z-score measures how many standard deviations the measure is from the mean. After finding the Z-score, we look at the z-score table and find the p-value associated with this z-score. This p-value is the probability that the value of the measure is smaller than X, that is, the percentile of X. Subtracting 1 by the pvalue, we get the probability that the value of the measure is greater than X.

Central Limit Theorem

The Central Limit Theorem estabilishes that, for a normally distributed random variable X, with mean [tex]\mu[/tex] and standard deviation [tex]\sigma[/tex], the sampling distribution of the sample means with size n can be approximated to a normal distribution with mean [tex]\mu[/tex] and standard deviation [tex]s = \frac{\sigma}{\sqrt{n}}[/tex].

For a skewed variable, the Central Limit Theorem can also be applied, as long as n is at least 30.

In this question, we have that:

[tex]\mu = 10.8, \sigma = 3.7, n = 35, s = \frac{3.7}{\sqrt{35}} = 0.6254[/tex]

What is the probability that the sample mean will be greater than 10 years?

This is 1 subtracted by the pvalue of Z when X = 10. So

[tex]Z = \frac{X - \mu}{\sigma}[/tex]

By the Central Limit Theorem

[tex]Z = \frac{X - \mu}{s}[/tex]

[tex]Z = \frac{10 - 10.8}{0.6254}[/tex]

[tex]Z = -1.28[/tex]

[tex]Z = -1.28[/tex] has a pvalue of 0.1003

10.03% probability that the sample mean will be greater than 10 years

Suppose we want to choose a value of x at least 5 units away from 14.A) Think about some values of x that meet this constraint.B) Write an inequality that represents all values of x that meet this constraint. Hint. no answer givenC) On the number line below, represent all values of x that meet this constraint. Clear All Draw: Line segments and raysClosed dotOpen dot

Answers

Answer:

(a) [9, 19]

(b)  [tex]9 \leq x \leq 19[/tex]

(c)See attachment

Step-by-step explanation:

We want to choose a value of x at least 5 units away from 14.

(a)Now, 14-5=9 and 14+5=19

The possible values of x ranges is in the closed interval [9,19]

(b) Since x is at least 5 units away from 14, we have:

[tex]|14-x|\leq5[/tex]

Solving the absolute inequality

[tex]-5 \leq 14-x \leq 5\\$In $ -5 \leq 14-x\\ x \leq 14+5\\x \leq 19\\\\$In $ 14-x \leq 5\\ 14-5 \leq x\\9 \leq x\\$Therefore,an inequality that represents all values of x that meet this constraint is:$\\9 \leq x \leq 19[/tex]

(c)To draw the number line, we use a closed dot since we have a less than or equal to sign.

Inequalities are used to represent unequal expressions.

The inequality that represents the scenario is [tex]\mathbf{|x-14| \ge 5}[/tex]Some possible values of x are: 5 and 20

Represent the value with x.

A value that is at least 5 from 14 can take any of the following forms:

[tex]\mathbf{14 -x \ge 5}[/tex]

[tex]\mathbf{x-14 \ge 5}[/tex]

The inequalities can be combined as:

[tex]\mathbf{|x-14| \ge 5}[/tex]

The values of x are solved as follows:

[tex]\mathbf{14 -x \ge 5}[/tex]

[tex]\mathbf{14 - 5 \ge x}[/tex]

[tex]\mathbf{9\ge x}[/tex]

Rewrite as:

[tex]\mathbf{x \le 9}[/tex]

Similarly, we have:

[tex]\mathbf{x-14 \ge 5}[/tex]

[tex]\mathbf{x \ge 14 + 5}[/tex]

[tex]\mathbf{x \ge 19}[/tex]

So, we have:

[tex]\mathbf{x \le 9}[/tex] or [tex]\mathbf{x \ge 19}[/tex]

This can be combined as:

[tex]\mathbf{9 \ge x \ u\ x \ge 19}[/tex]

Some possible values of x are: 5 and 20

Read more about inequalities at:

https://brainly.com/question/1930605

Conduct the following test at the alpha = 0.10 level of significance by determining ​(a) the null and alternative​ hypotheses, ​(b) the test​ statistic, and​ (c) the​ P-value. Assume that the samples were obtained independently using simple random sampling.
Test whether p1 not equals p2. Sample data are x1 = 28​, n1 = 254​, x2 = 38​, and n2 = 301.

Answers

Answer:

a) H0: p1 - p2 = 0

H1: p1 - p2 ≠ 0

b) z=-0.58

c) p-value = 0.562

Step-by-step explanation:

We need to determine whether p1 is not equals p2, so the null and alternative hypothesis are:

H0: p1 - p2 = 0

H1: p1 - p2 ≠ 0

Where p1 and p2 are the proportions of the population. Additionally, the proportions of the sample p1' and p2' are calculated as:

[tex]p1'=\frac{x1}{n1}=\frac{28}{254}=0.1102\\p2'=\frac{x2}{n2}=\frac{38}{301}=0.1262[/tex]

Then, the test statistic is calculated using the following equation:

[tex]z=\frac{(p1'-p2')-(p1-p2)}{\sqrt{p'(1-p')(\frac{1}{n1}+\frac{1}{n2})} }[/tex]

Where p' is calculated as:

[tex]p'=\frac{x1+x2}{n1+n2}=\frac{28+38}{254+301}=0.1189[/tex]

So, replacing the values, we get that the test statistic is:

[tex]z=\frac{(0.1102-0.1262)-(0)}{\sqrt{0.1189(1-0.1189)(\frac{1}{254}+\frac{1}{301})}}=-0.58[/tex]

Finally, using the standard normal table, the p-value is equal to:

[tex]p-value=2*P(z<-0.58)=2*0.281=0.562[/tex]

The p-value is greater that the value of alpha 0.1, so we can't reject the null hypothesis and there is evidence to said that p1 and p2 are equals.

Use Euler's method to obtain a four-decimal approximation of the indicated value. First use h = 0.1 and then use h = 0.05.
Find an explicit solution for the initial-value problem and then fill in the following tables. (Round your answers to four decimal places. Percentages may be rounded to two decimal places.) y' = 2xy, y(1) = 1; y(1.5) y(x) = (explicit solution)
h = 0.1
xn yn Actual Value Absolute Error % Rel. Error
1.00 1.0000 1.0000 0.0000 0.00
1.10 1.2337 1.20 1.5527 1.30 1.9937
1.40 2.6117 1.50 3.4903
h = 0.05
xn yn Actual Value Absolute Error % Rel. Error
1.00 1.0000 1.0000 0.0000 0.00
1.05 1.1000 1.1079 0.0079 0.71
1.10 1.2155 1.2337 0.0182 1.48
1.15 1.3492 1.3806 0.0314 2.27
1.20 1.5044 1.5527 0.0483 3.11
1.25 1.6849 1.7551 0.0702 4.00
1.30 1.8955 1.9937 0.0982 4.93
1.35 2.1419 2.2762 0.1343 5.90
1.40 2.4311 2.6117 0.1806 6.92
1.45 2.7715 3.0117 0.2402 7.98

Answers

Answer:

  see below for the tables

Step-by-step explanation:

The differential equation is separable, so the solution is ...

  [tex]\displaystyle\dfrac{dy}{dx}=2xy\\\\\int{\dfrac{dy}{y}}=\int{2x}\,dx\\\\\ln{y}=x^2+C\\\\\text{Considering the initial condition, $C=-1$}\\\\\boxed{y=e^{x^2-1}}[/tex]

__

The values for yn are y+y'·h = y+2xyh. We take the "absolute error" to be the (signed) difference between the calculated yn and the actual value y(x).

341 is 55% of what number? Round your answer to the nearest hundredth

Answers

Your answer will be 620!

Answer:

the number is 620.

Step-by-step explanation:

in other words the question is 55% of what number is 341?

let the number be Y.

[tex]\frac{55}{100}*Y=341[/tex]

[tex]\frac{55Y}{100}=341[/tex]

[tex]55Y=341*100[/tex]

[tex]55Y=34100[/tex]

[tex]Y=\frac{34100}{55}[/tex]

Y=620

Find the equation of the line.
(6,4) (-4,-5)

Answers

Answer:

y = (9 x)/10 - 7/5

Step-by-step explanation:

y-y/x-x

60 POINTS Explain the process and solve a logarithmic equation. log x + log 8 – 2log 4 = 7.

Answers

Answer:

[tex]x = 20000000[/tex]

Step-by-step explanation:

Recall the power property of logarithms which states:

[tex]log(a^n)=n\,\,log(a)[/tex]

to re-write [tex]2\,log(4)=log(4^2)=log(16)[/tex]

and then use the product and quotient rules of logarithms:

[tex]log (A*B)=log(A)+log(B)[/tex]

and

[tex]log (\frac{A}{B} )=log(A)-log(B)[/tex]

to rewrite the combination of logarithms on the left of the equal sign as a single logarithm:

[tex]log(x)+log(8)-2\,\,log(4)=7\\log(x)+log(8)-log(16)=7\\log(\frac{8\,x}{16}) =7\\log(\frac{x}{2}) =7[/tex]

and now re-write this equation in exponent form to get rid of the logarithm:

[tex]10^7=\frac{x}{2} \\2\,\,\,10^7 = x\\x = 20000000[/tex]

Find the area of the figure. (Use “sq units” as the units of measure) Hints: Break it down into the triangle and 2 rectangles. Pay attention to the measurements.

Answers

Answer:

Area of the figure is 60.75 m²

Step-by-step explanation:

If we divide the given figure in three parts,

Area of the figure = Area of rectangle (1) + Area of rectangle (2) + Area of triangle (3)

Area of rectangle (1) = (Length × width)

                                  = 3 × 4.5

                                  = 13.5 m²

Area of rectangle (2) = (12 × 3)

                                  = 36 m²

Area of triangle (3) = [tex]\frac{1}{2}(\text{Base})(\text{Height})[/tex]

                               = [tex]\frac{1}{2}(7.5)(3)[/tex]

                               = 11.25 m²

Area of the complete figure = 13.5 + 36 + 11.25

                                              = 60.75 m²

Therefore, area of the figure is 60.75 m²

Find 3 points that solve the equation -x + 2y= 2

Answers

Answer:

(0,1)

(-2,0)

(2,2)

Step-by-step explanation:

a television set was sold during a clearance sale for dollar 299.00the television was sold at a discount of 70 percent, what was the list price?

Answers

Answer:

  $996.67

Step-by-step explanation:

The discounted price is ...

  (list price)(100% -70%) = $299.00

  list price = $299.00/0.30 . . . . . divide by 0.30

  list price = $996.67

If the following inequality is divided by −2, what is the resulting inequality? −2x≤−20

Answers

Step-by-step explanation:

When dividing by a negative number, you "flip" the inequalities sign so you would divide -20 by -2 which would leave you with 10. X is left where it is and you flip the inequalities sign to a ≥ sign.

final answer =

x ≥ 10

What is the value of x?

Answers

Answer:

x=-14

Step-by-step explanation:

The 2 angles are opposite of each other. This means that they are vertical angles, are they are congruent.

Since they are congruent, we can set them equal to each and solve for x.

9x+184=7x+156

To solve the equation, we want to find out what x is. In order to do this, we have to get x by itself. Perform the opposite of what is being done to the equation. Keep in mind, everything done to one side, has to be done to the other.

First, subtract 7x from both sides.

9x-7x+184=7x-7x+156

9x-7x+184=156

2x+184=156

Next, subtract 184 from both sides.

2x+184-184=156-184

2x=156-184

2x=-28

Finally, divide both sides by 2.

2x/2= -28/2

x=-28/2

x= -14

The value of X is -14.

please see the attached picture for full solution

Hope it helps

Good luck on your assignment

Other Questions
who is better batman or superman from all of the movies Aball is thrown downward from a window in a tall building. Its position at time tin seconds is s = 16t ^ 2 + 32t , where s is in feet. How long to the nearest will the ball to fall 276 ? Which of the following structure is a protective features of the Agama lizardto terrestrial habitat? (A) claw (B) Gular fold (c) scaly skin (D) nuchal crest (R) Tail A random sample of 1285 residents from rural and urban areas were surveyed about their opinion about using daylight savings time. Below is the gathered data. Assuming theres no relationship between residential area and opinion on daylight savings time, how many people who live in a rural area would you expect to be in favor of continuing to use daylight savings time? Please round up to a whole number. Stop using Daylight Savings Time Continue using Daylight Savings Time Total Rural 341 281 622Urban 353 310 663Total 694 591 1285 Pearson's Chi-square test X-squared 0.32 df= 1 p-value ? A truck was purchased for $120,000 and it was estimated to have a $24,000 salvage value at the end of its useful life. Monthly depreciation expense of $2,000 was recorded using the straight-line method. The annual depreciation rate is Assume the spot rate on the Canadian dollar is C$1.1847. The risk-free nominal rate in the U.S. is 5 percent while it is only 4 percent in Canada. What one-year forward rate will create interest rate parity? 2) An employee makes a job change. Her original salary was $65,000, and her salary at her new job is $54,000. What percent decrease was her salary change?Please show your work for full credit. At September 30, the accounts of East Terrace Medical Center (ETMC) include the following: Accounts Receivable $143,000 Allowance for Uncollectible Accounts (credit balance) 3,200 During October through December, ETMC completed the following selected transactions: Dec. 28 Wrote off as uncollectible the $1,500 account receivable from Brown Co. and the $1,100 account receivable from Jacob Weiss. Dec. 31 Recorded uncollectible-account expense based on the again of accounts receivable, which follows: Age of AccountsAccounts Receivable 1-30 Days 31-60 Days 61-90 Days Over 90 Days $163,000 $100,000 $40,000 $ 14,000 $9,000 Estimated percent uncollectible 0.1% 1% 10% 30% 1. Record the transactions in the journal. 2. Open the Allowance for Uncollectible Accounts, and post entries affecting that account. Keep a running balance.3. Show how East Terrace Medical Center should report accounts receivable on its balance sheet at December 31, 2007 How does "Madam and the Rent Man" reflect itsbistorical setting? Explain your answer. giving brainliestt pls help Find the value of x in this polygon No trecho "Tirei do bolso uma nota de cinquenta lhe ofereci pela sua franqueza. " o termo grifado introduz uma conjuno:A) aditiva B) adversativaC) alternativaD) explicativaOBS: o que est grifado a letra E, e por favor botem uma explicao alm da resposta. I need help with this pls Solve for f.7f 9 = 9 4ff = Dans les phrases suivantes, donne la nature des mots souligns : Where does the body absorb most nutrients? The expression (2x + 1)4 is expanded and simplified. Which monomial listed below is a term in the result? 8x3 12x3 32x3 48x3 Sentence combining ( combine it into one sentence) A. My best friend was sitting on a train. C. The train was bound for Paris. B. I was sitting on a train. D. This is when the nightmare began. C. The train was in Frankfurt. distinguish between random sampling and systematic sampling with example . Pls Fastt How do the details in this timeline support the authors'purpose?ation ofcoloniesns, noabolishedtoThe details about the shift of power in France helppersuade readers that France was important to the sugarindustry.The details about the changing laws in France help informreaders that Napoleon wanted to produce sugar cheaplyby using enslaved people.The details about the French Revolution help informreaders about the process of executing enemies.The details about Marie Antoinette help persuade readersthat the French royalty were not well liked.cutedpniesorth Which of the following is not a reason that power plants are unable to achieve 100 percent efficiency? A. The amount of work obtained from a process never has work needing to go into it. B. Not all energy is converted heat from chemical reactions into useful work. C. Entropy increases. D. Heat is often a waste product